3
$\begingroup$

Let $\{v_i\}_{i=1}^m \in \mathbb C^n$ be vectors of norm $1$ so that $F = \max_{i \neq j} \vert v_i^\dagger v_j\vert$ is small. Is it possible to show that $$\lambda_\max(S) = 1 + O(1)mF^2$$ where $$S = \sum_{i=1}^m v_i^\dagger v_i$$ For $F$ close to $1$ it is not too hard to show that $\lambda_\max(S) \leq 1 + mF$, but I am hoping to show that as $F$ gets small the bound can be tightened. To show the $1 + mF$ bound, one simply applies Cauchy-Schwarz once, but the regime where Cauchy-Schwarz is tight is the same regime where $F$ is close to $1$. When $F$ is small the application of Cauchy-Schwarz is not tight and so there should be a way to do better...

$\endgroup$

1 Answer 1

1
$\begingroup$

I assume $v_i$ are linearly independent and I believe you wanted to write

$$S = \sum_{i=1}^m v_i v_i^\dagger $$

Let $x$ be an eigenvector of $S$ for a nonzero eigenvalue, so that $Sx = \lambda x, \lambda \neq 0$. Therefore, necessarily $x \in \operatorname{Im} S$. Let $$ x = \sum_{i=1}^m \alpha_i v_i $$ for some $\alpha_i$. So,

$$Sx = \sum_{i=1}^m v_i v_i^\dagger x = \sum_{i=1}^m v_i \left( \sum_{j=1}^m \alpha_j v_i^\dagger v_j \right) = \lambda \sum_{i=1}^m \alpha_i v_i $$

So, for all $i$ we have $$\lambda \alpha_i = \sum_{j=1}^m \alpha_j v_i^\dagger v_j$$

Select $i$ such that $|\alpha_i| = \max_j |\alpha_j|$. Therefore, $$ \begin{align*} | \lambda | | \alpha_i | &\leq \sum_{j=1}^m | \alpha_j | | v_i^\dagger v_j | \\ &= | \alpha_i | + \sum_{j \neq i} |\alpha_j| | v_i^\dagger v_j | \\ &\leq | \alpha_i | \left( 1+ (m-1) F \right) \end{align*}$$

So, we improved the bound as $|\lambda| \leq 1+ (m-1) F$.

We can also show that this bound is the tightest possible by constructing $S$ such that $\lambda = 1+ (m-1) F$. Note that equality occurs if $\alpha_i = \alpha_j$ and $F = v_i^\dagger v_j$ for all $i\neq j$. We can select $m$ linearly independent vectors of norm 1 such that $F = v_i^\dagger v_j$ for all $i\neq j$ for arbitrary $|F|<1$. In this case if $\alpha_i = 1, \forall i$, then $x$ is an eigenvector of $S$. So, we have equality. By the way, for $m=2$ we always have equality for any $v_1,v_2$ since $x=v_1+v_2$ is an eigenvector of $S$ with $\lambda = 1 + v_1^\dagger v_2$.

$\endgroup$

You must log in to answer this question.

Start asking to get answers

Find the answer to your question by asking.

Ask question

Explore related questions

See similar questions with these tags.